This paper was converted on www.awesomepapers.org from LaTeX by an anonymous user.
Want to know more? Visit the Converter page.

LINEAR RELATIONS AMONG GALOIS CONJUGATES OVER 𝔽q​(t)\mathbb{F}_{q}(t)

Will Hardt and John Yin

1. Abstract

We classify the coefficients (a1,…,an)βˆˆπ”½q​[t]n(a_{1},...,a_{n})\in\mathbb{F}_{q}[t]^{n} that can appear in a linear relation βˆ‘i=1nai​γi=0\sum_{i=1}^{n}a_{i}\gamma_{i}=0 among Galois conjugates Ξ³iβˆˆπ”½q​(t)Β―\gamma_{i}\in\overline{\mathbb{F}_{q}(t)}. We call such an nn-tuple a Smyth tuple. Our main theorem gives an affirmative answer to a function field analogue of a 1986 conjecture of Smyth [4] over β„š\mathbb{Q}. Smyth showed that certain local conditions on the aia_{i} are necessary and conjectured that they are sufficient. Our main result is that the analogous conditions are necessary and sufficient over 𝔽q​(t)\mathbb{F}_{q}(t), which we show using a combinatorial characterization of Smyth tuples from [4]. We also formulate a generalization of Smyth’s Conjecture in an arbitrary number field that is not a straightforward generalization of the conjecture over β„š\mathbb{Q} due to a subtlety occurring at the archimedean places.

2. Introduction

The question of how prevalent linear relations among Galois conjugates are has been studied from multiple angles. In [2] it is shown that for a Hilbertian field KK and all but finitely many nonnegative integers nn, there exists an algebraic number α∈KΒ―\alpha\in\overline{K} of degree 2n​n!2^{n}n! whose conjugates span a vector space of dimension nn. In these cases, the dimension of relations between conjugates is 2n​n!βˆ’n2^{n}n!-n, and so in this sense, linear relations among conjugates are plentiful.

In an older paper, Smyth showed [4, Cor. 2] that for any a1,…,anβˆˆβ„€a_{1},\dots,a_{n}\in{\mathbb{Z}} with g​c​d​(a1,…,an)=1gcd(a_{1},\dots,a_{n})=1, if there exist Galois conjugates Ξ³1,…,Ξ³nβˆˆβ„šΒ―\gamma_{1},\dots,\gamma_{n}\in\overline{\mathbb{Q}} such that βˆ‘i=1nai​γi=0\sum_{i=1}^{n}a_{i}\gamma_{i}=0, then the aia_{i} necessarily satisfy the following two properties.

  1. (1)

    |ai|β‰€βˆ‘jβ‰ i|aj||a_{i}|\leq\sum_{j\neq i}|a_{j}| for all ii

  2. (2)

    Every prime pp divides at most nβˆ’2n-2 of the aia_{i}.

Smyth conjectured that the converse is true as well over β„š\mathbb{Q}.

Conjecture 2.1 (Smyth’s Conjecture).

[4] If (a1,…,an)βˆˆβ„šn(a_{1},\dots,a_{n})\in\mathbb{Q}^{n} satisfy (1) and (2), then there exist Galois conjugates Ξ³1,…,Ξ³n\gamma_{1},\dots,\gamma_{n} so that βˆ‘i=1nai​γi=0\sum_{i=1}^{n}a_{i}\gamma_{i}=0.

A natural way to generalize (1) and (2) to an arbitrary field KK is as follows.

  • (1’)

    For any archimedean absolute value |β‹…||\cdot| of KK, we have |ai|β‰€βˆ‘jβ‰ i|aj||a_{i}|\leq\sum_{j\neq i}|a_{j}| for all ii.

  • (2’)

    For any nonarchimedean absolute value |β‹…||\cdot| of KK, we have |ai|≀maxjβ‰ i⁑|aj||a_{i}|\leq\max_{j\neq i}|a_{j}| for all ii.

We call (1’) and (2’) the absolute value criteria over KK.

Note that for any a∈Kβˆ—a\in K^{*}, a tuple (a1,…,an)∈Kn(a_{1},...,a_{n})\in K^{n} satisfies the absolute value criteria if and only if (a​a1,…,a​an)(aa_{1},...,aa_{n}) does, and similarly, (a1,…,an)(a_{1},...,a_{n}) is a Smyth tuple if and only if (a​a1,…,a​an)(aa_{1},...,aa_{n}) is a Smyth tuple. Hence, when KK is the field of fractions of a principal ideal domain, it is enough to look at coprime tuples (a1,…,an)(a_{1},...,a_{n}).

Conjecture 2.1 remains open. Our main result is Theorem 4.2, which answers in the affirmative the natural analogue of Smyth’s question over 𝔽q​(t){\mathbb{F}}_{q}(t). We will also formulate in Conjecture 10.1 a generalization of Conjecture 2.1 for arbitrary number fields, which involves a subtlety not present in the cases of β„š\mathbb{Q} or 𝔽q​(t)\mathbb{F}_{q}(t). A MathOverflow post of David Speyer [1] proves a special case of this conjecture.

In Section 3 we record some terminology that we will use throughout the paper. In Section 4 we set up Theorem 4.2, which we go on to prove in Section 5. Section 6 examines how close the Galois relations constructed in our proof of Theorem 4.2 are to β€œas small as possible.” In Section 7 we show that the conjugates in our constructed linear relations for Smyth triples can be chosen to have the full symmetric group as their Galois group and in Section 8 we give heuristic reasoning for why we should expect the Galois group of linear Galois relations to be large in general. In Section 9, we record some results from Section 6 that also apply over β„š\mathbb{Q}. Finally, in Section 10, we formulate a number field generalization of Conjecture 2.1, and then reduce it slightly to Conjecture 10.7.

3. Preliminaries

We lay out some terminology and a background result.

We say that Ξ³1,…,Ξ³n\gamma_{1},...,\gamma_{n} are Galois conjugates over K if they are all roots of the same irreducible polynomial over KK. (When the base field KK is clear from the context, we will often omit it from our terminology.) Throughout the paper, we assume Galois conjugates are nonzero.

A linear Galois relation will mean a linear relation βˆ‘i=1nai​γi=0\sum_{i=1}^{n}a_{i}\gamma_{i}=0 among Galois conjugates Ξ³1,…,Ξ³n∈KΒ―\gamma_{1},...,\gamma_{n}\in\overline{K} with coefficients ai∈Ka_{i}\in K.

We will call a tuple (a1,…,an)βˆˆπ”½q​[t]n(a_{1},...,a_{n})\in\mathbb{F}_{q}[t]^{n} coprime if a1,…,ana_{1},...,a_{n} generate the unit ideal (and similarly for (a1,…,an)βˆˆβ„€n(a_{1},...,a_{n})\in\mathbb{Z}^{n}). The (logarithmic) height of a coprime tuple (a1,…,an)βˆˆπ”½q​[t]n(a_{1},...,a_{n})\in\mathbb{F}_{q}[t]^{n} is maxi⁑deg⁑(ai)\max_{i}\deg(a_{i}). The (logarithmic) height of a coprime tuple (a1,…,an)βˆˆβ„€n(a_{1},...,a_{n})\in\mathbb{Z}^{n} is maxi⁑log⁑|ai|\max_{i}\log|a_{i}|. A coprime tuple is said to be a Smyth tuple if its coordinates appear as the coefficients of a linear Galois relation.

A balanced multiset of tuples – which we will sometimes shorten to β€œbalanced multiset” – with respect to a tuple (a1,…,an)(a_{1},...,a_{n}) is a nonempty collection of nonzero solutions {(xi1,…,xin)∈Kn}i=1N\{(x_{i_{1}},...,x_{i_{n}})\in K^{n}\}_{i=1}^{N} to the equation βˆ‘i=1nai​xi=0\sum_{i=1}^{n}a_{i}x_{i}=0 such that the multiset {xij}i=1N\{x_{i_{j}}\}_{i=1}^{N} is independent of jj; in other words, for all x∈Kx\in K, xx appears in each of the NN coordinate positions the same number of times.

A balanced multiset {(xi1,…,xin)∈Kn}i=1N\{(x_{i_{1}},...,x_{i_{n}})\in K^{n}\}_{i=1}^{N} is a 1-factor if the multiset {xi1}i=1N\{x_{i_{1}}\}_{i=1}^{N} is in fact a set. (To see the justification for this terminology, consider the hypergraph whose vertices are elements of KK and whose edges are ordered tuples (x1,…,xn)(x_{1},...,x_{n}) of solutions to βˆ‘i=1nai​xi=0\sum_{i=1}^{n}a_{i}x_{i}=0. Then a 1-factor in this hypergraph is precisely our definition of 1-factor.)

The degree of a linear Galois relation βˆ‘i=1nai​γi=0\sum_{i=1}^{n}a_{i}\gamma_{i}=0 will refer to the degree [K​(Ξ³1):K][K(\gamma_{1}):K] of the conjugates in the linear relation. Similarly the Galois group of a linear Galois relation βˆ‘i=1nai​γi=0\sum_{i=1}^{n}a_{i}\gamma_{i}=0 will mean Gal⁑(E/K)\operatorname{Gal}(E/K), where EE is the Galois closure of K​(Ξ³1)K(\gamma_{1}).

Following standard terminology, we will call a Galois extension L/KL/K a GG-extension if Gal⁑(L/K)β‰…G\operatorname{Gal}(L/K)\cong G.

Our final preliminary is the fact that over any Hilbertian field, there exist SdS_{d}-extensions for all dβ‰₯1d\geq 1. This is a special case of [2, Prop. 8]. We will apply this fact to the Hilbertian field 𝔽q​(t)\mathbb{F}_{q}(t).

4. Smyth’s Conjecture over 𝔽q​(t){\mathbb{F}}_{q}(t)

Although our primary focus in this section (and this paper) is the function field 𝔽q​(t)\mathbb{F}_{q}(t), we will often state definitions and theorems over an arbitrarily field KK when it is not substantially more complicated to do so.

Our approach in this paper benefits from having several equivalent notions of Smyth tuples. Most importantly, Smyth proved a combinatorial characterization of Smyth tuples involving balanced multisets. We record this, along with one other characterization of Smyth tuples, in Proposition 4.1.

Proposition 4.1.

Let KK be any field such that there exist SdS_{d}-extensions of KK for all dβ‰₯1d\geq 1. The following are equivalent for (a1,…,an)∈Kn(a_{1},...,a_{n})\in K^{n}.

  1. (1)

    (a1,…,an)(a_{1},...,a_{n}) is a Smyth tuple

  2. (2)

    There exists a balanced multiset of tuples with respect to (a1,…,an)(a_{1},...,a_{n})

  3. (3)

    There exist permutation matrices X1,…,XnX_{1},...,X_{n} such that det(βˆ‘i=1nai​Xi)=0\det(\sum_{i=1}^{n}a_{i}X_{i})=0

Proof.

Smyth proved (1) ⇔\iff (2) for K=β„šK=\mathbb{Q} [4, Thm. 2], and the proof works for any KK satisfying the hypotheses of this theorem.

(2) ⟹\implies (3): Let T={(xi​1,xi​2,…,xi​n)}i=1NT=\{(x_{i1},x_{i2},...,x_{in})\}_{i=1}^{N} be a balanced multiset of tuples of size NN. For j:1≀j≀nj:1\leq j\leq n, let vj=(xi​j)i=1Nv_{j}=(x_{ij})_{i=1}^{N} be the vector in KNK^{N} obtained by taking the jt​hj^{th} entry from each tuple in TT. By definition of balanced multiset, there exist (not necessarily unique) NΓ—NN\times N permutation matrices X1,…,XnX_{1},...,X_{n} such that Xi​vn=viX_{i}v_{n}=v_{i} for all ii. Thus (βˆ‘i=1nai​Xi)​vn=βˆ‘i=1nai​vi=0(\sum_{i=1}^{n}a_{i}X_{i})v_{n}=\sum_{i=1}^{n}a_{i}v_{i}=0, so βˆ‘i=1nai​Xi\sum_{i=1}^{n}a_{i}X_{i} has nontrivial kernel.

(3) ⟹\implies (2): Reverse the previous argument as follows. Let vnv_{n} be any nonzero vector in the kernel of βˆ‘i=1nai​Xi\sum_{i=1}^{n}a_{i}X_{i} and let vi:=Xi​vnv_{i}:=X_{i}v_{n} for 1≀i≀n1\leq i\leq n. Then the coordinates of the vectors v1,…,vnv_{1},...,v_{n} give a balanced multiset of tuples as above. ∎

Our main theorem is the following.

Theorem 4.2.

Let nβ‰₯3n\geq 3 be an integer. A coprime tuple (a1,…,an)βˆˆπ”½q​[t]n(a_{1},...,a_{n})\in\mathbb{F}_{q}[t]^{n} is a Smyth tuple if and only if a1,…,ana_{1},\dots,a_{n} satisfy the absolute value criteria over 𝔽q​(t){\mathbb{F}}_{q}(t).

The forward direction of Theorem 4.2 was established by Smyth over β„š\mathbb{Q} in a proof that is valid over a general field KK. We record this result in Lemma 4.3.

Lemma 4.3.

[4, Cor. 2] Let KK be any field. If (a1,…,an)∈Kn(a_{1},...,a_{n})\in K^{n} is a Smyth tuple, then a1,…,ana_{1},...,a_{n} satisfy the absolute value criteria over KK.

5. Proof of Theorem 4.2

We now turn toward the proof of our main result, Theorem 4.2, which will follow essentially as a corollary from Proposition 5.1.

Let VNβŠ‚π”½q​[t]V_{N}\subset{\mathbb{F}}_{q}[t] be the set of polynomials of degree <N<N. We will show that, surprisingly, for any coprime (a1,..,an)βˆˆπ”½q[t]n(a_{1},..,a_{n})\in\mathbb{F}_{q}[t]^{n} satisfying the absolute value criteria and any Nβ‰₯d=max⁑degi⁑aiN\geq d=\max\deg_{i}a_{i}, the set of all solutions (x1,…,xn)∈VNn(x_{1},...,x_{n})\in V_{N}^{n} to the equation βˆ‘i=1nai​xi=0\sum_{i=1}^{n}a_{i}x_{i}=0 is a balanced set.

Proposition 5.1.

Let nβ‰₯3n\geq 3 be an integer. Let (a1,…,an)βˆˆπ”½q​[t]n(a_{1},\dots,a_{n})\in{\mathbb{F}}_{q}[t]^{n} of height dd be a coprime tuple satisfying the absolute value criteria. Let Nβ‰₯dN\geq d be an integer and let jj be an integer so that 1≀j≀n1\leq j\leq n. Fix xj∈VNx_{j}\in V_{N}. Then the number of tuples (x1,…,xjβˆ’1,xj+1,…​xn)∈VNnβˆ’1(x_{1},\dots,x_{j-1},x_{j+1},\dots x_{n})\in V_{N}^{n-1} satisfying βˆ‘i=1nai​xi=0\sum_{i=1}^{n}a_{i}x_{i}=0 is qN​(nβˆ’2)βˆ’dq^{N(n-2)-d}. In particular, this count does not depend on jj.

Proof.

Without loss of generality, we let j=1j=1. By the absolute value criteria, the maximum degree of a1,…,ana_{1},\dots,a_{n} is achieved at least twice. Hence, some aia_{i} with iβ‰ 1i\neq 1 has degree dd; without loss of generality, assume that ana_{n} does. Let c=a1​x1c=a_{1}x_{1}. Define

S={(x2,…,xn)∈VNnβˆ’1:c+βˆ‘i=2nai​xi=0}S=\{(x_{2},\dots,x_{n})\in V_{N}^{n-1}:c+\sum_{i=2}^{n}a_{i}x_{i}=0\}

Our goal is to compute #​S\#S. To do so, we will project onto 𝔽q​[t]/an{\mathbb{F}}_{q}[t]/a_{n}, so we define

SΒ―={(x2Β―,…,xnβˆ’1Β―)∈(𝔽q​[t]/an)nβˆ’2:cΒ―+βˆ‘i=2nβˆ’1ai​xiΒ―=0}\overline{S}=\{(\overline{x_{2}},\dots,\overline{x_{n-1}})\in({\mathbb{F}}_{q}[t]/a_{n})^{n-2}:\overline{c}+\sum_{i=2}^{n-1}\overline{a_{i}x_{i}}=0\}

Reducing modulo ana_{n} in each coordinate and throwing out the last coordinate gives a surjective q(Nβˆ’d)​(nβˆ’2)q^{(N-d)(n-2)}-to-1 map Sβ†’SΒ―S\to\overline{S}; the pre-image of any (xΒ―2,…,xΒ―nβˆ’1)∈SΒ―(\overline{x}_{2},...,\overline{x}_{n-1})\in\overline{S} is

{(x2+h2​an,…,xnβˆ’1+hnβˆ’1​an,βˆ’(c+βˆ‘i=2nβˆ’1ai​xian+βˆ‘i=2nβˆ’1ai​hi)):hi∈VNβˆ’d},\left\{\left(x_{2}+h_{2}a_{n},...,x_{n-1}+h_{n-1}a_{n},-\left(\frac{c+\sum_{i=2}^{n-1}a_{i}x_{i}}{a_{n}}+\sum_{i=2}^{n-1}a_{i}h_{i}\right)\right):h_{i}\in V_{N-d}\right\},

where xix_{i} is the unique polynomial of degree <d<d equal to xΒ―i\overline{x}_{i} mod ana_{n}. Thus, we have #​S=q(Nβˆ’d)​(nβˆ’2)​#​SΒ―\#S=q^{(N-d)(n-2)}\#\overline{S}.

So we want to count the number of solutions (xΒ―2,…​xΒ―nβˆ’1)∈(𝔽q​[t]/(an))nβˆ’2(\overline{x}_{2},...\overline{x}_{n-1})\in({\mathbb{F}}_{q}[t]/(a_{n}))^{n-2} to cΒ―+βˆ‘i=2nβˆ’1ai​xiΒ―=0\overline{c}+\sum_{i=2}^{n-1}\overline{a_{i}x_{i}}=0. Let an=∏pjeja_{n}=\prod p_{j}^{e_{j}} be the prime factorization of ana_{n}. Let Rj:=𝔽q​[t]/(pjej)R_{j}:={\mathbb{F}}_{q}[t]/(p_{j}^{e_{j}}); by the Chinese Remainder Theorem, it will suffice to count the number of solutions in RjR_{j} for each jj.

Specifically, let SΒ―j={(x2Β―,…,xΒ―nβˆ’1)∈Rjnβˆ’2:cΒ―+βˆ‘i=2nβˆ’1ai¯​xiΒ―=0}\overline{S}_{j}=\{(\overline{x_{2}},...,\overline{x}_{n-1})\in R_{j}^{n-2}:\bar{c}+\sum_{i=2}^{n-1}\overline{a_{i}}\overline{x_{i}}=0\}; then the Chinese Remainder Theorem implies that

#​SΒ―=∏j#​SΒ―j.\#\overline{S}=\prod_{j}\#\overline{S}_{j}.

We now compute #​SΒ―j\#\overline{S}_{j}. Recall that by the absolute value criteria, for all jj, there are at least two aia_{i} that are not divisible by pjp_{j}. Of course ana_{n} is divisible by all pjp_{j}; hence, for all jj, there is at least one aia_{i}, with 1<i<n1<i<n, such that pj∀aip_{j}\nmid a_{i}, in which case aiΒ―\overline{a_{i}} is a unit in RjR_{j}. Thus, we can write xiΒ―=cΒ―+βˆ‘kβ‰ i,kβ‰ 1ak​xkΒ―aiΒ―\overline{x_{i}}=\frac{\overline{c}+\sum_{k\neq i,k\neq 1}\overline{a_{k}x_{k}}}{\overline{a_{i}}}, and so any collection of choices of xkΒ―\overline{x_{k}} for k∈{2,…,nβˆ’1}βˆ–{i}k\in\{2,...,n-1\}\setminus\{i\}, will give a unique choice of xiΒ―\overline{x_{i}}. There are #​Rj=qdeg⁑(pjej)\#R_{j}=q^{\deg(p_{j}^{e_{j}})} choices for each xΒ―k\overline{x}_{k}, so #​SΒ―j=qdeg⁑(pjej)​(nβˆ’3)\#\overline{S}_{j}=q^{\deg(p_{j}^{e_{j}})(n-3)}. Thus, since βˆ‘jdeg⁑(pjej)=deg⁑(an)=d\sum_{j}\deg(p_{j}^{e_{j}})=\deg(a_{n})=d, we have

#​S=q(Nβˆ’d)​(nβˆ’2)β€‹βˆjqdeg⁑(pjej)​(nβˆ’3)=q(Nβˆ’d)​(nβˆ’2)​qd​(nβˆ’3)=qN​(nβˆ’2)βˆ’d,Β as desired.\#S=q^{(N-d)(n-2)}\prod_{j}q^{\deg(p_{j}^{e_{j}})(n-3)}=q^{(N-d)(n-2)}q^{d(n-3)}=q^{N(n-2)-d},\text{ as desired.}

∎

Now Theorem 4.2 follows easily.

Proof of Theorem 4.2.

(β‡’\Rightarrow): This was Lemma 4.3.

(⇐\Leftarrow): Without loss of generality, we assume (a1,…,an)(a_{1},\dots,a_{n}) is a coprime tuple in 𝔽q​[t]n{\mathbb{F}}_{q}[t]^{n}. Let TNT_{N} be the set of all tuples (x1,…,xn)∈VNn(x_{1},\dots,x_{n})\in V_{N}^{n} satisfying βˆ‘i=1nai​xi=0\sum_{i=1}^{n}a_{i}x_{i}=0 and enumerate TN={(xi​1,…,xi​n)}i=1tT_{N}=\{(x_{i1},\dots,x_{in})\}_{i=1}^{t} where t=|TN|t=|T_{N}|. In Proposition 5.1 we showed that for every x∈VNx\in V_{N} and all i:1≀i≀ni:1\leq i\leq n, the number of tuples (x1,…,xn)(x_{1},\dots,x_{n}) in TNT_{N} with xi=xx_{i}=x is qN​(nβˆ’2)βˆ’dq^{N(n-2)-d}. This means that for each j:1≀j≀nj:1\leq j\leq n, the multiset {(xi​j)}i=1t\{(x_{ij})\}_{i=1}^{t} is precisely qN​(nβˆ’2)βˆ’dq^{N(n-2)-d} copies of VNV_{N}. Thus, TNT_{N} is a balanced (multi)set of tuples. So by Proposition 4.1, (a1,…,an)(a_{1},\dots,a_{n}) is a Smyth tuple. ∎

Remark 5.2.

By setting N=d=maxi⁑deg⁑(ai)N=d=\max_{i}\deg(a_{i}) and n=3n=3 in Proposition 5.1, we see that if (a1,a2,a3)(a_{1},a_{2},a_{3}) is a Smyth triple, then TdT_{d} is a 1-factor.

6. Degree of Galois Conjugates in Linear Galois Relations

In the previous section, we produced balanced multisets for all Smyth tuples, i.e. all tuples for which it is possible to find a balanced multiset. We relied on Proposition 4.1 to turn the balanced multisets into linear Galois relations. In this section, we consider the question of how close these linear Galois relations are to β€œas small as possible.” As a first attempt to formulate this question, we might ask for a uniform lower bound β„“n​(D)\ell_{n}(D) on the degree of all linear Galois relations whose coefficients are an nn-tuple of height DD. However, this is quickly seen to be an uninteresting question, as there exist Smyth tuples of arbitrarily large height that are coefficients of a bounded degree linear Galois relation. (For instance, if βˆ‘i=1nΟ΅i​ai=0\sum_{i=1}^{n}\epsilon_{i}a_{i}=0 where Ο΅i∈{1,βˆ’1}\epsilon_{i}\in\{1,-1\}, then there is a balanced multiset with respect to (a1,…,an)(a_{1},...,a_{n}) of size 22.) So instead, we ask for a lower bound Ln​(D)L_{n}(D) on the β€œworst” Smyth nn-tuple of height DD.

Definition 6.1.

Let LnK​(D)L^{K}_{n}(D) be the minimal LL so that there is some (a1,…,an)∈Kn(a_{1},...,a_{n})\in K^{n} of height DD such that every linear Galois relation with coefficients a1,…,ana_{1},...,a_{n} has degree at least LL

In this section the underlying field KK will always be 𝔽q​(t)\mathbb{F}_{q}(t), and so we will omit it from the notation. In Section 9 we will record analogous results about L3β„šβ€‹(D)L^{\mathbb{Q}}_{3}(D).

In Corollary 6.4, we prove that there exist Smyth triples (a1,a2,a3)(a_{1},a_{2},a_{3}) of height DD for which one cannot find a balanced multiset of size less than |VDβˆ–{0}|=qDβˆ’1|V_{D}\setminus\{0\}|=q^{D}-1. In other words, besides removing zero from the multiset, our construction in Section 4 cannot be uniformly improved upon.

In Proposition 6.2, we (non-constructively) show that a 33-term linear Galois relation of degree dd with coefficients (a1,a2,a3)(a_{1},a_{2},a_{3}) implies the existence of a balanced multiset with respect to (a1,a2,a3)(a_{1},a_{2},a_{3}) of size dd. Thus we obtain in Corollary 6.5 the lower bound qDβˆ’1≀L3​(D)q^{D}-1\leq L_{3}(D).

Note that this does not imply that the linear Galois relations arising from our balanced multisets in the n=3n=3 case have degree as small as possible; the linear Galois relations arising from a balanced multiset of size qDβˆ’1q^{D}-1 may have degree as large as (qDβˆ’1)!(q^{D}-1)! (see Theorem 7.2).

6.1. A Map of Representations

We will use the framework of representation theory to prove Proposition 6.2. We now introduce in a general setting the representations we will be working with.

Let KK be a field and let L/KL/K be a Galois extension with Galois group GG. Let γ∈L\gamma\in L be any element and let VG​a​l​C​o​n​jV_{GalConj} be the KK-vector space spanned by the conjugates of Ξ³\gamma. Then VG​a​l​C​o​n​jV_{GalConj} is a GG-invariant subspace of LL, and so the action of GG on LL as a KK-vector space gives an action of GG on VG​a​l​C​o​n​jV_{GalConj}. We call this the Galois conjugate representation.

Suppose that Ξ³\gamma has degree dd over KK. Let Ξ³1,…,Ξ³d\gamma_{1},\dots,\gamma_{d} be the Galois conjugates of Ξ³\gamma. Let VP​e​r​mV_{Perm} be a dd-dimensional vector space over KK with basis Ξ±1,…,Ξ±d\alpha_{1},...,\alpha_{d} and define an action of GG on VP​e​r​mV_{Perm} by gβ‹…Ξ±i=Ξ±jg\cdot\alpha_{i}=\alpha_{j} whenever g​(Ξ³i)=Ξ³jg(\gamma_{i})=\gamma_{j}. This is the permutation representation. We will write (VP​e​r​m,ρP​e​r​m)(V_{Perm},\rho_{Perm}) and (VG​a​l​C​o​n​j,ρG​a​l​C​o​n​j)(V_{GalConj},\rho_{GalConj}) to denote the permutation and Galois conjugate representations of GG, respectively.

Notice that specializing the formal variables Ξ±i\alpha_{i} to the Galois conjugates Ξ³i\gamma_{i} gives a surjective map Ο•\phi of GG-representations from (VP​e​r​m,ρP​e​r​m)(V_{Perm},\rho_{Perm}) to (VG​a​l​C​o​n​j,ρG​a​l​C​o​n​j)(V_{GalConj},\rho_{GalConj}).

Proposition 6.2.

Let K=𝔽q​(t)K={\mathbb{F}}_{q}(t). Suppose that (a1,…,an)βˆˆπ”½q​[t]n(a_{1},...,a_{n})\in\mathbb{F}_{q}[t]^{n} is a coprime tuple and that Ξ³=Ξ³1,…,Ξ³n\gamma=\gamma_{1},...,\gamma_{n} are Galois conjugates over KK such that βˆ‘i=1nai​γi=0\sum_{i=1}^{n}a_{i}\gamma_{i}=0. Let m:=[K(Ξ³):K]m:=[K(\gamma):K]. Then there exists a balanced multiset of tuples with respect to (a1,…,an)(a_{1},...,a_{n}) of size mm.

Proof.

Let E/KE/K be the Galois closure of K​(Ξ³1,…,Ξ³n)K(\gamma_{1},...,\gamma_{n}), and write G:=Gal⁑(E/K)G:=\operatorname{Gal}(E/K). Let Ξ³=Ξ³1\gamma=\gamma_{1} and choose g1=1,…,gn∈Gg_{1}=1,...,g_{n}\in G so that gi​γ=Ξ³ig_{i}\gamma=\gamma_{i}. Notice that βˆ‘i=1nai​gi\sum_{i=1}^{n}a_{i}g_{i} gives a linear transformation on both VP​e​r​mV_{Perm} and VG​a​lV_{Gal}, which we’ll call ψP​e​r​m\psi_{Perm} and ψG​a​l\psi_{Gal} respectively. Moreover, they commute with the map of GG-representations Ο•\phi, i.e. the following diagram commutes.

VP​e​r​m{V_{Perm}}VG​a​l​C​o​n​j{V_{GalConj}}VP​e​r​m{V_{Perm}}VG​a​l​C​o​n​j{V_{GalConj}}ψP​e​r​m\scriptstyle{\psi_{Perm}}Ο•\scriptstyle{\phi}ψG​a​l​C​o​n​j\scriptstyle{\psi_{GalConj}}Ο•\scriptstyle{\phi}

By assumption there is a nonzero γ∈ker⁑ψG​a​l​C​o​n​j\gamma\in\ker\psi_{GalConj} and a simple diagram-chasing argument shows that ψP​e​r​m\psi_{Perm} has nontrivial kernel too. Therefore det(βˆ‘i=1nai​gi)=0\det(\sum_{i=1}^{n}a_{i}g_{i})=0 on VP​e​r​mV_{Perm}, and the construction from Proposition 4.1 yields a balanced multiset of tuples of size mm.

∎

Smyth also produced balanced multisets of tuples from linear Galois relations in [4, Thm. 2]. Given a linear Galois relation βˆ‘i=1nai​γi=0\sum_{i=1}^{n}a_{i}\gamma_{i}=0, Smyth’s argument yielded balanced multisets of size the degree of the smallest normal extension β„šβ€‹(Ξ²){\mathbb{Q}}(\beta) of β„š{\mathbb{Q}} containing Ξ³\gamma so that [β„š(Ξ²):β„š(Ξ³)]β‰₯n[{\mathbb{Q}}(\beta):{\mathbb{Q}}(\gamma)]\geq n. Our argument shows the existence of a smaller balanced set, of size [β„šβ€‹(Ξ³):β„š][{\mathbb{Q}}(\gamma):{\mathbb{Q}}].

Proposition 6.3.

Let (a,b,c)βˆˆπ”½q​[t]3(a,b,c)\in\mathbb{F}_{q}[t]^{3} be a coprime triple. Any nonzero balanced set with respect to (a,b,c)(a,b,c) (if one exists) has size as least the order of βˆ’a/b-a/b in (𝔽q​[t]/c)βˆ—(\mathbb{F}_{q}[t]/c)^{*}.

Proof.

Let T={(t1​j,t2​j,t3​j)}j=1mT=\{(t_{1j},t_{2j},t_{3j})\}_{j=1}^{m} be any balanced multiset of triples with respect to (a,b,c)(a,b,c). Without loss of generality, suppose that the t1​jt_{1j} (j=1,2,…,mj=1,2,...,m) are coprime polynomials. Choose kk so that t1​kt_{1k} is not divisible by cc.

For a polynomial xβˆˆπ”½q​[t]x\in\mathbb{F}_{q}[t], we will use the notation xΒ―\overline{x} to denote the image of xx under the mod-c quotient map 𝔽q​[t]→𝔽q​[t]/c\mathbb{F}_{q}[t]\to\mathbb{F}_{q}[t]/c. From the equation a​t1​k+b​t2​k+c​t3​k=0at_{1k}+bt_{2k}+ct_{3k}=0, we have t2​kΒ―=(βˆ’a/b)​t1​kΒ―\overline{t_{2k}}=\overline{(-a/b)t_{1k}}. By balancedness, this shows that there is some jj so that t1​jΒ―=(βˆ’a/b)​t1​kΒ―\overline{t_{1j}}=\overline{(-a/b)t_{1k}}.

Iterating this argument, we see that {(βˆ’a/b)n​t1​kΒ―:nβˆˆβ„€+}βŠ‚{t1​jΒ―}j=1m\{\overline{(-a/b)^{n}t_{1k}}:n\in\mathbb{Z}^{+}\}\subset\{\overline{t_{1j}}\}_{j=1}^{m}, and the result follows. ∎

In the case where cc is irreducible and βˆ’a/b-a/b is a generator for (𝔽q​[t]/c)βˆ—({\mathbb{F}}_{q}[t]/c)^{*}, we get the following.

Corollary 6.4.

Let (a,b,c)βˆˆπ”½q​[t]3(a,b,c)\in{\mathbb{F}}_{q}[t]^{3} be a Smyth triple. Suppose that cc is irreducible with deg⁑(c)=dβ‰₯deg⁑(b),deg⁑(a)\deg(c)=d\geq\deg(b),\deg(a) and also that (βˆ’a/b)(-a/b) is a generator for (𝔽q​[t]/c)βˆ—({\mathbb{F}}_{q}[t]/c)^{*}. Then the smallest balanced multiset of triples with respect to (a,b,c)(a,b,c) has size qdβˆ’1q^{d}-1.

Proof.

From Proposition 5.1, setting N=dN=d and n=3n=3, and removing the triples (0,0,0)(0,0,0), we obtain a balanced multiset (in fact set) of triples of size qdβˆ’1q^{d}-1.

On the other hand, Proposition 6.3 shows that this there cannot be a smaller balanced multiset. ∎

Corollary 6.5.

L3​(D)β‰₯qDβˆ’1L_{3}(D)\geq q^{D}-1.

Proof.

By Proposition 6.2, it suffices to show that for all DD, there is some triple (a,b,c)(a,b,c) with respect to which every balanced multiset of tuples has size at least qDβˆ’1q^{D}-1. In other words, we are reduced to constructing a triple satisfying the hypotheses of Corollary 6.4 with d=Dd=D.

Let b,cb,c be distinct irreducible polynomials of degree DD. Let g∈(𝔽q​[t]/c)βˆ—g\in(\mathbb{F}_{q}[t]/c)^{*} be any multiplicative generator and let aa be the unique polynomial of degree ≀Dβˆ’1\leq D-1 such that a=βˆ’g​ba=-gb (mod c). Then a,b,ca,b,c are pairwise coprime and their maximum degree is achieved twice, so they are a Smyth triple by Theorem 4.2. Thus they satisfy the hypotheses of Corollary 6.4. ∎

7. Galois Group of Constructed Linear Galois Relations

In this section we show that the conjugates in our construction for Smyth triples from Section 5 can be chosen to have the full symmetric group as their Galois group.

We begin with a lemma.

Lemma 7.1.

Let LL be an SdS_{d}-extension of K:=𝔽q​(t)K:=\mathbb{F}_{q}(t). There exist conjugates Ξ±1,…,Ξ±d\alpha_{1},\dots,\alpha_{d} with βˆ‘i=1dΞ±iβ‰ 0\sum_{i=1}^{d}\alpha_{i}\neq 0 such that L=K​(Ξ±1,…,Ξ±d)L=K(\alpha_{1},\dots,\alpha_{d}).

Proof.

Choose α∈L\alpha\in L of degree dd with conjugates Ξ±=Ξ±1,…,Ξ±d\alpha=\alpha_{1},...,\alpha_{d} such that L=K​(Ξ±1,…,Ξ±d)L=K(\alpha_{1},...,\alpha_{d}). We will show that βˆ‘i=1dΞ±iβ‰ 0\sum_{i=1}^{d}\alpha_{i}\neq 0 for some 1≀k≀d1\leq k\leq d. Let pk=βˆ‘i=1dxikp_{k}=\sum_{i=1}^{d}x_{i}^{k} be the kt​hk^{th} power sum polynomial. Suppose for contradiction that pk​(Ξ±1,…,Ξ±d)=0p_{k}(\alpha_{1},\dots,\alpha_{d})=0 for all kβ‰₯1k\geq 1. Let eie_{i} be the it​hi^{th} elementary symmetric polynomial and let f​(x)=βˆ‘i=0dai​xif(x)=\sum_{i=0}^{d}a_{i}x^{i} be the minimal polynomial of Ξ±1\alpha_{1}. We have that ek​(Ξ±1,…,Ξ±d)=(βˆ’1)k​adβˆ’ke_{k}(\alpha_{1},\dots,\alpha_{d})=(-1)^{k}a_{d-k} for 1≀k≀d1\leq k\leq d.

Since we assume pi​(Ξ±1,…,Ξ±d)=0p_{i}(\alpha_{1},\dots,\alpha_{d})=0 for all 1≀i≀d1\leq i\leq d, the Newton identities give

(βˆ’1)kβˆ’1​k​ek​(Ξ±1,…,Ξ±d)=0(-1)^{k-1}ke_{k}(\alpha_{1},\dots,\alpha_{d})=0

So, ek​(Ξ±1,…,Ξ±d)=0e_{k}(\alpha_{1},\dots,\alpha_{d})=0 for all p∀kp\nmid k. Thus, the minimal polynomial of Ξ±1\alpha_{1} is inseparable, contradicting the fact that Ξ±\alpha generates a separable extension. So, there is some kk for which βˆ‘i=1dΞ±ikβ‰ 0\sum_{i=1}^{d}\alpha_{i}^{k}\neq 0. If βˆ‘i=1dΞ±i=0\sum_{i=1}^{d}\alpha_{i}=0, then k>1k>1 and the Ξ±ik\alpha_{i}^{k} are not guaranteed to generate LL. But we can find some nonzero c∈Kc\in K for which Ξ±1+c​α1k\alpha_{1}+c\alpha_{1}^{k} and its conjugates generate LL. Indeed, for all but finitely many c∈Kc\in K, the elements Ξ±i+c​αik\alpha_{i}+c\alpha_{i}^{k} are pairwise distinct, in which case no nontrivial KK-automorphisms of LL fix the field K​(Ξ±i+c​αik)i=1dK(\alpha_{i}+c\alpha_{i}^{k})_{i=1}^{d}. Since βˆ‘i=1d(Ξ±1+c​α1k)=cβ€‹βˆ‘i=1dΞ±1kβ‰ 0\sum_{i=1}^{d}(\alpha_{1}+c\alpha_{1}^{k})=c\sum_{i=1}^{d}\alpha_{1}^{k}\neq 0 for nonzero cc, we are done.

∎

Theorem 7.2.

Let K=𝔽q​(t)K=\mathbb{F}_{q}(t) and fix an integer nβ‰₯3n\geq 3. Let (a1,…,an)βˆˆπ”½q​[t]n(a_{1},\dots,a_{n})\in\mathbb{F}_{q}[t]^{n} be a coprime tuple. Suppose that there is a 1-factor T={(xi1,…,xin)∈Kn}i=1mT=\{(x_{i_{1}},...,x_{i_{n}})\in K^{n}\}_{i=1}^{m} with respect to (a1,…,an)(a_{1},...,a_{n}) of size mm. Then there exist Galois conjugates Ξ³1,…,Ξ³d\gamma_{1},\dots,\gamma_{d} such that βˆ‘i=1nai​γi=0\sum_{i=1}^{n}a_{i}\gamma_{i}=0 and Gal⁑(K​(Ξ³1)/K)=Sm\operatorname{Gal}(K(\gamma_{1})/K)=S_{m}.

Proof.

By Lemma 7.1, we obtain Ξ±1,…,Ξ±d\alpha_{1},\dots,\alpha_{d} such that K​(Ξ±1,…,Ξ±d)K(\alpha_{1},\dots,\alpha_{d}) is an SdS_{d}-extension of KK and βˆ‘i=1mΞ±iβ‰ 0\sum_{i=1}^{m}\alpha_{i}\neq 0. Define Ξ³1:=βˆ‘j=1mv1,j​αj\gamma_{1}:=\sum_{j=1}^{m}v_{1,j}\alpha_{j}. Note that since SS is a 1-factor, the entries v1,jv_{1,j} (j=1,2,…,m)(j=1,2,...,m) are distinct. We will write vjv_{j} for v1,jv_{1,j}. If K​(Ξ³1)K(\gamma_{1}) is a proper subextension of LL, then it must be fixed by a nontrivial element Οƒ\sigma of SmS_{m}. So suppose that

βˆ‘j=1mvj​αj=Ξ³1=σ​(Ξ³1)=βˆ‘j=1mvΟƒβˆ’1​(j)​αj.\sum_{j=1}^{m}v_{j}\alpha_{j}=\gamma_{1}=\sigma(\gamma_{1})=\sum_{j=1}^{m}v_{\sigma^{-1}(j)}\alpha_{j}.

Then βˆ‘j=1m(vjβˆ’vΟƒβˆ’1​(j))​αj=0\sum_{j=1}^{m}(v_{j}-v_{\sigma^{-1}(j)})\alpha_{j}=0. By [4, Thm 13.4.2], which is presented as a lemma about β„š{\mathbb{Q}} but is valid for KK as well, the coefficients vjβˆ’vΟƒβˆ’1​(j)v_{j}-v_{\sigma^{-1}(j)} must all be equal. Then for c:=v1βˆ’vσ​(1)c:=v_{1}-v_{\sigma(1)}, we have cβ€‹βˆ‘j=1mΞ±j=0c\sum_{j=1}^{m}\alpha_{j}=0. By assumption, βˆ‘j=1mΞ±jβ‰ 0\sum_{j=1}^{m}\alpha_{j}\neq 0, so c=0c=0. Therefore Οƒ=1\sigma=1, and we have proved that Ξ³1\gamma_{1} is not contained in a proper subextension of K​(Ξ³1,…,Ξ³d)K(\gamma_{1},...,\gamma_{d}).

∎

8. A Heuristic for the Existence of a Linear Galois Relation with Prescribed Galois Group

We just saw in Section 7 that the conjugates appearing in the linear Galois relations we constructed for Smyth triples (a,b,c)βˆˆπ”½q​[t]3(a,b,c)\in\mathbb{F}_{q}[t]^{3} can be chosen to generate an SqdS_{q^{d}}-extension of 𝔽q​(t)\mathbb{F}_{q}(t) where dd is the height of (a,b,c)(a,b,c). In this section, we consider which Galois groups we expect to arise in this context.

Definition 8.1.

If (a1,…,an)(a_{1},...,a_{n}) is a Smyth tuple, GβŠ‚SmG\subset S_{m} is a finite group, and there exist Galois conjugates Ξ³1,…,Ξ³n\gamma_{1},...,\gamma_{n} over KK such that βˆ‘i=1nai​γi=0\sum_{i=1}^{n}a_{i}\gamma_{i}=0 and Gal⁑(K​(Ξ³1)/K)β‰…G\operatorname{Gal}(K(\gamma_{1})/K)\cong G, then we say (a1,…,an)(a_{1},...,a_{n}) is a Smyth GG-tuple.

Question 8.2.

Suppose that (a1,…,an)βˆˆπ”½q​[t]n(a_{1},...,a_{n})\in\mathbb{F}_{q}[t]^{n} is a Smyth tuple. For which m>0m>0 and transitive subgroups GβŠ‚SmG\subset S_{m} is (a1,…,an)(a_{1},...,a_{n}) a Smyth GG-tuple?

Heuristic 8.3, which depends only on |G||G| and mm, will correctly predict that Smyth tuples (a1,…,an)(a_{1},...,a_{n}) are always SqNS_{q^{N}}-tuples for sufficiently large NN. (Recall that Remark 5.2 and Theorem 7.2 together show that every Smyth triple is an SqNS_{q^{N}}-triple for sufficiently large NN.)

By Proposition 4.1, a coprime nn-tuple (a1,…,an)βˆˆπ”½q​[t]n(a_{1},...,a_{n})\in\mathbb{F}_{q}[t]^{n} of height dd is a Smyth tuple if and only if there exist permutation matrices X1,…,XnX_{1},...,X_{n} such that det(βˆ‘i=1nai​Xi)=0\det(\sum_{i=1}^{n}a_{i}X_{i})=0. Our approach so far has been to look for balanced multisets, which is essentially asking, given (a1,…,an)(a_{1},...,a_{n}), whether or not there exists a vector vβˆˆπ”½q​[t]nv\in\mathbb{F}_{q}[t]^{n} and permutation matrices X1,..XnX_{1},..X_{n} such that (βˆ‘i=1nai​Xi)​v=0(\sum_{i=1}^{n}a_{i}X_{i})v=0. We now shift our perspective slightly by fixing both the tuple (a1,…,an)(a_{1},...,a_{n}) and a vector vv, and asking whether or not there exist such permutation matrices in GG. Since vv will have no repeated entries, the heuristic is suited only to detect 1-factors, as opposed to a general balanced multiset.

In particular, we will fix a vector vNβˆˆπ”½q​[t]qNv_{N}\in\mathbb{F}_{q}[t]^{q^{N}} whose coordinates are all the elements of VNV_{N} in some order, each occurring exactly once. We have already seen that for any Smyth triple (a1,a2,a3)(a_{1},a_{2},a_{3}), there do exist permuation matrices X1,X2,X3∈SqNX_{1},X_{2},X_{3}\in S_{q^{N}} such that (a1​X1+a2​X2+a3​X3)​vN=0(a_{1}X_{1}+a_{2}X_{2}+a_{3}X_{3})v_{N}=0 (Remark 5.2). Our heuristic, in the case of triples, suggests an answer to the question of whether this remains true if we insist on choosing X1,X2,X3X_{1},X_{2},X_{3} from some subgroup GβŠ‚SqNG\subset S_{q^{N}}.

Heuristic 8.3.

Fix NN and enumerate the set of polynomials over 𝔽q\mathbb{F}_{q} of degree <N<N: VN={f1,f2,…,fqN}V_{N}=\{f_{1},f_{2},...,f_{q^{N}}\}, and fix a vector v=(fi)i=1qNβˆˆπ”½q​[t]qNv=(f_{i})_{i=1}^{q^{N}}\in\mathbb{F}_{q}[t]^{q^{N}} whose coordinates are all the elements of VNV_{N}, each occurring exactly once. Let Xn=IX_{n}=I be the qNΓ—qNq^{N}\times q^{N} identity matrix. Then choose random permutations X1,…,Xnβˆ’1∈GβŠ‚SqNX_{1},...,X_{n-1}\in G\subset S_{q^{N}} and assume that for each j:1≀j≀qNj:1\leq j\leq q^{N}, the sum βˆ‘i=1nai​vXiβˆ’1​(j)\sum_{i=1}^{n}a_{i}v_{X_{i}^{-1}(j)} takes values in VN+dV_{N+d} uniformly and independently at random.

Remark 8.4.

A shortcoming of Heuristic 8.3 is that it doesn’t see the (necessary) β€œlocal” conditions – it would predict that any nn-tuple (a1,…,an)(a_{1},...,a_{n}), even those which are not Smyth tuples, is a Smyth GG-tuple for large enough GG.

Remark 8.5.

The reason for fixing Xn=IX_{n}=I is that if βˆ‘i=1nai​Xi​v=0\sum_{i=1}^{n}a_{i}X_{i}v=0, then also βˆ‘i=1nai​(X​Xi)​v=0\sum_{i=1}^{n}a_{i}(XX_{i})v=0 for every X∈GX\in G. This means that we are really concerned with GG-orbits of nn-tuples of GG, and we identify these with (nβˆ’1)(n-1)-tuples of elements of GG by fixing the last coordinate.

The result of this assumption is that we have βˆ‘i=1n(ai​Xi)​v=0\sum_{i=1}^{n}(a_{i}X_{i})v=0 with probability (qβˆ’(N+d))qN(q^{-(N+d)})^{q^{N}}. On the other hand, we have |G|nβˆ’1|G|^{n-1} choices for X1,…,Xnβˆ’1X_{1},...,X_{n-1}, so, according to this model, the probability of failing to find X1,…,XnX_{1},...,X_{n} such that βˆ‘i=1n(ai​Xi)​v=0\sum_{i=1}^{n}(a_{i}X_{i})v=0 is pN:=(1βˆ’qβˆ’(N+d)​qN)|G|(nβˆ’1)p_{N}:=(1-q^{-(N+d)q^{N}})^{|G|^{(n-1)}}.

This probability goes to 0 as Nβ†’βˆžN\to\infty for |G|=Ο‰q,d​(q(N+d)​qN/(nβˆ’1))|G|=\omega_{q,d}(q^{(N+d)q^{N}/(n-1)}) (Proposition 8.6). (We are using the standard Landau asymptotic notation here, in which f​(n)=ω​(g​(n))f(n)=\omega(g(n)) means that limnβ†’βˆžg​(n)f​(n)=0\lim_{n\to\infty}\frac{g(n)}{f(n)}=0.)

Notably, if we plug in n=2n=2, we see that Heuristic 8.3 correctly does not predict that a random pair (a1,a2)(a_{1},a_{2}) is a Smyth pair; indeed, one can check that (a1,a2)βˆˆβ„€2(a_{1},a_{2})\in\mathbb{Z}^{2} is a Smyth pair if and only if a1=Β±a2a_{1}=\pm a_{2}.

Proposition 8.6.

Write |GN|=cN​(q(N+d)​qN)1/(nβˆ’1)|G_{N}|=c_{N}(q^{(N+d)q^{N}})^{1/(n-1)} and suppose that limNβ†’βˆžcN=∞\lim_{N\to\infty}c_{N}=\infty. Then limNβ†’βˆžpN=0\lim_{N\to\infty}p_{N}=0.

By Stirling’s formula, Heuristic 8.3 suggests an affirmative answer to Question 8.2 for G=SqNG=S_{q^{N}} when nβ‰₯3n\geq 3, in accordance with the fact that we found linear Galois relations among triples of conjugates which generate SqNS_{q^{N}}-extensions. It also predicts an affirmative answer for the alternating groups AqNA_{q^{N}}, but predicts negative answers for the cyclic group of order qNq^{N} and the dihedral group of order 2​qN2q^{N}.

9. Smyth’s Conjecture Over β„š\mathbb{Q}

In this brief section we note that some of our results from Section 6 can be easily translated from 𝔽q​(t)\mathbb{F}_{q}(t) over to β„š\mathbb{Q}. In particular, we have the following analogues of Proposition 6.3, Corollary 6.4, and Corollary 6.5.

Proposition 9.1.

If (a,b,c)βˆˆβ„€3(a,b,c)\in{\mathbb{Z}}^{3} then any balanced multiset of triples with respect to (a,b,c)(a,b,c) (if one exists) has size at least the order of βˆ’a/b-a/b in (β„€/c​℀)βˆ—(\mathbb{Z}/c\mathbb{Z})^{*}.

The proof of this proposition is virtually identical to the proof of Proposition 6.3. We also obtain analogous corollaries.

Corollary 9.2.

Let (a,b,c)βˆˆβ„€3(a,b,c)\in\mathbb{Z}^{3} be a coprime triple. Suppose that cc is prime and that βˆ’a/b-a/b is a generator for (β„€/c​℀)βˆ—(\mathbb{Z}/c\mathbb{Z})^{*}. Then the smallest balanced multiset of triples with respect to (a,b,c)(a,b,c) (if one exists) has size at least |(β„€/c​℀)βˆ—|=cβˆ’1|(\mathbb{Z}/c\mathbb{Z})^{*}|=c-1.

Corollary 9.3.

Let pDp_{D} denote the largest prime at most eDe^{D}. Assuming Conjecture 2.1, we have L3β„šβ€‹(D)β‰₯pDβˆ’1L^{\mathbb{Q}}_{3}(D)\geq p_{D}-1 for Dβ‰₯5D\geq 5, and consequently, L3β„šβ€‹(D)=Ω​(eD)L_{3}^{\mathbb{Q}}(D)=\Omega(e^{D}).

Proof.

Similar to Corollary 6.5, we just have to construct a fitting Smyth triple. Assuming the Smyth Conjecture, for (a,b,c)βˆˆβ„€3(a,b,c)\in{\mathbb{Z}}^{3} to be a Smyth triple is the same as for a,b,ca,b,c to be pairwise coprime and satisfy the triangle inequalities (a+bβ‰₯ca+b\geq c, a+cβ‰₯ba+c\geq b, b+cβ‰₯ab+c\geq a).

Assume that Dβ‰₯5D\geq 5 and let pDp_{D} be the largest prime at most eDe^{D} and let g∈(β„€/pD​℀)βˆ—g\in({\mathbb{Z}}/p_{D}{\mathbb{Z}})^{*} be any multiplicative generator other than βˆ’1modpD-1\mod p_{D}.

Take nn to be the representative of βˆ’1g+1modpD\frac{-1}{g+1}\mod p_{D} between 0 and pDβˆ’1p_{D}-1. Then, βˆ’n+1n≑gmodpD-\frac{n+1}{n}\equiv g\mod p_{D} and if n+(n+1)β‰₯pDn+(n+1)\geq p_{D}, then (n,n+1,pD)(n,n+1,p_{D}) satisfy the absolute value criteria over β„š\mathbb{Q}. (Since we are assuming Conjecture 2.1, this is equivalent to being a Smyth triple.) On the other hand, if n+(n+1)<pDn+(n+1)<p_{D}, then (pDβˆ’(n+1),pDβˆ’n,pD)(p_{D}-(n+1),p_{D}-n,p_{D}) satisfies the absolute value criteria. In either case, the inequality now follows from Proposition 9.1.

By the prime number theorem, for any k<1k<1, we have pN>k​Np_{N}>kN for large enough NN. Therefore pDβˆ’1=Ω​(eD)p_{D}-1=\Omega(e^{D}).

∎

10. Smyth’s Conjecture Over Number Fields

Recall that in any field, the absolute value criteria are necessary conditions for being a Smyth tuple (Lemma 4.3). We showed in Theorem 4.2 that these criteria are sufficient for being a Smyth tuple over 𝔽q​(t)\mathbb{F}_{q}(t), and Smyth conjectured the same over β„š\mathbb{Q} (Conjecture 2.1).

However, an example presented by David Speyer in a MathOverflow post shows that the absolute value criteria are not sufficient for being a Smyth tuple in a general number field [1]. In particular, the triple (1,1,1+βˆ’152)(1,1,\frac{1+\sqrt{-15}}{2}) satisfies the absolute value criteria, but is not a Smyth triple. Note that this triple achieves equality in the archimedean absolute value inequalities.

Speyer showed in the same post that for triples of the form (1,1,a3)(1,1,a_{3}), if one amends the absolute value criteria to be strict inequalities for the archimedean absolute values, then they become a sufficient condition for being a Smyth triple. On the other hand, examples such as (2,3,βˆ’5)(2,3,-5) show that we cannot simply amend the archimedean absolute value criteria to be strict inequalities, as (2,3,βˆ’5)(2,3,-5) trivially is a Smyth triple. Instead, if some analogue of Smyth’s Conjecture is true in number fields, it must be a little more sensitive to the cases in which there is equality in one of the archimedean absolute value criteria.

In order to formulate what we think the right conjecture is, we define the strong absolute value criteria over KK as follows.

  • (1”)

    For any archimedean absolute value |β‹…||\cdot| of KK, we have |ai|<βˆ‘jβ‰ i|aj||a_{i}|<\sum_{j\neq i}|a_{j}| for all ii.

  • (2”)

    For any nonarchimedean absolute value |β‹…||\cdot| of KK, we have |ai|≀maxjβ‰ i⁑|aj||a_{i}|\leq\max_{j\neq i}|a_{j}| for all ii.

The strong absolute value criteria are obtained from the absolute value criteria by making the archimedean inequalities strict.

We are now ready to formulate our generalization of Conjecture 2.1.

Conjecture 10.1.

Let KK be a number field and π’ͺK{\mathcal{O}}_{K} its ring of integers. (a1,…,an)∈π’ͺKn(a_{1},...,a_{n})\in{\mathcal{O}}_{K}^{n} is a Smyth tuple if and only if (a1,…,an)(a_{1},...,a_{n}) satisfy the strong absolute value criteria over KK or there exist roots of unity Ο‰1,…,Ο‰n\omega_{1},...,\omega_{n} in some extension of KK such that βˆ‘i=1nai​ωi=0\sum_{i=1}^{n}a_{i}\omega_{i}=0.

Remark 10.2.

The K=β„šK=\mathbb{Q} case of Conjecture 10.1 is equivalent to Conjecture 2.1.

We will show in Proposition 10.6 that Conjecture 10.1 correctly deals with the tuples in which equality is achieved in one of the archimedean absolute value criteria. In particular, if (a1,…,an)(a_{1},...,a_{n}) is a tuple such that equality in one of the archimedean absolute value criteria, then any tuple in a balanced multiset with respect to (a1,…,an)(a_{1},...,a_{n}) (if one exists) is a scalar multiple of a tuple of roots of unity.

But first we need two lemmas, the first of which shows that the property of being a Smyth tuple is preserved by multiplying the coordinates by (possibly different) roots of unity.

Lemma 10.3.

Let (a1,…,an)∈π’ͺKn(a_{1},...,a_{n})\in{\mathcal{O}}_{K}^{n}. If Ο‰1,…,Ο‰n\omega_{1},...,\omega_{n} are roots of unity in some extension of KK and (a1,…,an)(a_{1},...,a_{n}) is a Smyth tuple, then (Ο‰1​a1,…,Ο‰n​an)(\omega_{1}a_{1},...,\omega_{n}a_{n}) is a Smyth tuple in π’ͺK​(Ο‰1,…,Ο‰n)n{\mathcal{O}}_{K(\omega_{1},\dots,\omega_{n})}^{n}.

Proof.

Without loss of generality we may assume that Ο‰2=…=Ο‰n=1\omega_{2}=...=\omega_{n}=1, as we can make the following argument about each coordinate in turn. Denote Ο‰:=Ο‰1\omega:=\omega_{1} and L:=K​(Ο‰)L:=K(\omega). Suppose that Ο‰m=1\omega^{m}=1.

Let {(xi1,…,xin)∈Kn}i=1N\{(x_{i_{1}},...,x_{i_{n}})\in K^{n}\}_{i=1}^{N} be a balanced multiset with respect to (a1,…,an)(a_{1},...,a_{n}). Then ⋃k=0mβˆ’1{(Ο‰kβˆ’1​xi1,Ο‰k​xi2,…,Ο‰k​xin)∈Ln}i=1N\bigcup_{k=0}^{m-1}\{(\omega^{k-1}x_{i_{1}},\omega^{k}x_{i_{2}},...,\omega^{k}x_{i_{n}})\in L^{n}\}_{i=1}^{N} is a balanced multiset with respect to (ω​a1,a2,…,an)(\omega a_{1},a_{2},...,a_{n}). ∎

Remark 10.4.

In particular, Lemma 10.3 shows that if there are roots of unity Ο‰1,…,Ο‰n\omega_{1},...,\omega_{n} such that βˆ‘i=1nai​ωi=0\sum_{i=1}^{n}a_{i}\omega_{i}=0, then (a1,…,an)(a_{1},...,a_{n}) is a Smyth tuple. Linear relations among roots of unity are a well-studied topic, going back at least to the 1960s. There are several results constraining the prevalence of such relations, indicating that such coefficients represent quite a small subset of Smyth tuples. A survey of some of these results is given in [5]. For instance, when a1,…,ana_{1},...,a_{n} are rational, a result of Mann gives an explicit upper bound depending only on nn for the order of the roots of unity Ο‰i\omega_{i} occurring in a minimal relation βˆ‘i=1nai​ωi=0\sum_{i=1}^{n}a_{i}\omega_{i}=0 [3]. (Here minimality means that no nonempty proper sub-sum vanishes, and that the equation is normalized so that Ο‰1=1\omega_{1}=1.)

Lemma 10.5.

Let (a1,…,an)∈π’ͺKn(a_{1},...,a_{n})\in{\mathcal{O}}_{K}^{n}. Suppose that there exists an archimedean absolute value |β‹…|Ξ½|\cdot|_{\nu} of KK and some ii for which |ai|Ξ½=βˆ‘jβ‰ i|aj|Ξ½|a_{i}|_{\nu}=\sum_{j\neq i}|a_{j}|_{\nu}. If there exists a balanced multiset with respect to (a1,…,an)(a_{1},...,a_{n}), then there exists a balanced multiset {(yi​1,…,yi​n)∈Kn}i=1N\{(y_{i1},...,y_{in})\in K^{n}\}_{i=1}^{N} with respect to (a1,…,an)(a_{1},...,a_{n}) whose coordinates yi​jy_{ij} all satisfy |yi​j|Ξ½=1|y_{ij}|_{\nu}=1.

Proof.

Without loss of generality assume that |a1|Ξ½=βˆ‘j>1|aj|Ξ½|a_{1}|_{\nu}=\sum_{j>1}|a_{j}|_{\nu}. Let S={(xi​1,…,xi​n)∈Kn}i=1NS=\{(x_{i1},...,x_{in})\in K^{n}\}_{i=1}^{N} be a balanced multiset with respect to (a1,…,an)(a_{1},...,a_{n}). Let X={xi​j:1≀i≀N,1≀j≀n}X=\{x_{ij}:1\leq i\leq N,1\leq j\leq n\} be the set of all coordinates appearing in SS. Write M=maxx∈X⁑|x|Ξ½M=\max_{x\in X}|x|_{\nu}. Any reference to β€œabsolute value” in this proof refers to |β‹…|Ξ½|\cdot|_{\nu}.

We claim that if a tuple in SS has a coordinate of absolute value MM, then all coordinates of that tuple have absolute value MM. To see this, first suppose that |xi0​1|=M|x_{i_{0}1}|=M for some i0i_{0}. Along with the assumptions that βˆ‘j=1naj​xi0​j=0\sum_{j=1}^{n}a_{j}x_{i_{0}j}=0 and |a1|Ξ½=βˆ‘j>2|aj|Ξ½|a_{1}|_{\nu}=\sum_{j>2}|a_{j}|_{\nu}, this implies that |xi0​j|Ξ½=M|x_{i_{0}j}|_{\nu}=M for all j=1,…,nj=1,...,n. What we’ve shown so far is that if the first coordinate in a tuple in SS has absolute value MM, then all coordinates in that tuple do.

But SS is balanced, which means that the multiset of first coordinates is the same as the multiset of jt​hj^{th} coordinates for every j=1,2,…,nj=1,2,...,n. In particular, each of these multisets has the same number of elements of absolute value MM, with the same multiplicities. Therefore coordinates of absolute value MM can only occur in tuples whose first coordinate has absolute value MM, and the claim is proved.

Thus the tuples whose coordinates have absolute value MM form a balanced sub-multiset of SS, and dividing all of these coordinates by an element of KK of absolute value MM, we obtain the desired balanced multiset. ∎

Proposition 10.6.

Let (a1,…,an)∈π’ͺKn(a_{1},...,a_{n})\in{\mathcal{O}}_{K}^{n}. Suppose that there exists an archimedean absolute value |β‹…|Ξ½|\cdot|_{\nu} of KK and some ii for which |ai|Ξ½=βˆ‘jβ‰ i|aj|Ξ½|a_{i}|_{\nu}=\sum_{j\neq i}|a_{j}|_{\nu}. Then (a1,…,an)(a_{1},...,a_{n}) is a Smyth tuple if and only if there exist roots of unity Ο‰1,…,Ο‰n\omega_{1},...,\omega_{n} (not necessarily in KK) such that βˆ‘i=1nai​ωi=0\sum_{i=1}^{n}a_{i}\omega_{i}=0.

Proof.

(⇐\Leftarrow): By assumption, (Ο‰1​a1,…,Ο‰n​an)(\omega_{1}a_{1},...,\omega_{n}a_{n}) is a Smyth tuple. The result now follows from Lemma 10.3.

(β‡’\Rightarrow): Let Ο•:Kβ†ͺβ„‚\phi:K\hookrightarrow\mathbb{C} be an embedding corresponding to the archimedean absolute value |β‹…|Ξ½|\cdot|_{\nu} and let ψ:β„šΒ―β†ͺβ„‚\psi:\overline{\mathbb{Q}}\hookrightarrow\mathbb{C} be an embedding of the algebraic closure of β„š\mathbb{Q} which extends Ο•\phi. We will write |β‹…||\cdot| for the standard absolute value of complex numbers. Without loss of generality assume that |ϕ​(a1)|=βˆ‘j>1|ϕ​(aj)||\phi(a_{1})|=\sum_{j>1}|\phi(a_{j})|.

By Lemma 10.5, there exists a balanced multiset S={(xi​1,…,xi​n)∈Kn}i=1NS=\{(x_{i1},...,x_{in})\in K^{n}\}_{i=1}^{N} with respect to (a1,…,an)(a_{1},...,a_{n}) such that all |xi​j|Ξ½=1|x_{ij}|_{\nu}=1. By definition of balanced multiset, we have

(10.1) βˆ‘j=1naj​xi​j=0\sum_{j=1}^{n}a_{j}x_{ij}=0

Now (10.1) along with |ϕ​(a1)|=βˆ‘j>1|ϕ​(aj)||\phi(a_{1})|=\sum_{j>1}|\phi(a_{j})| and the assumption that |ϕ​(xi​j)|=1|\phi(x_{ij})|=1 implies that

(10.2) arg⁑ϕ​(aj​xi​j)=Ο€+arg⁑ϕ​(a1​xi​1)​(mod ​2​π),Β for all ​i,j​ with ​j>1.\arg\phi(a_{j}x_{ij})=\pi+\arg\phi(a_{1}x_{i1})(\text{mod }2\pi),\text{ for all }i,j\text{ with }j>1.

In words, (10.2) is saying that given a fixed ii, the ϕ​(aj​xi​j)\phi(a_{j}x_{ij}) all β€œpoint in the same direction” for j>1j>1, and ϕ​(a1​xi​1)\phi(a_{1}x_{i1}) β€œpoints in the opposite direction.”

The rest of the argument is most easily expressed in polar coordinates. For all jj, let ϕ​(aj)=rj​θj\phi(a_{j})=r_{j}\theta_{j} where rjβˆˆβ„β‰₯0r_{j}\in\mathbb{R}^{\geq 0} and |ΞΈj|=1|\theta_{j}|=1. Fix any i0∈{1,2,..,N}i_{0}\in\{1,2,..,N\} and any j∈{2,…,n}j\in\{2,...,n\}. Then by (10.2) and the fact that all |ϕ​(xi​j)|=1|\phi(x_{ij})|=1, we have ϕ​(xi0​j)=βˆ’ΞΈ1ΞΈj​ϕ​(xi0​1)\phi(x_{{i_{0}}j})=-\frac{\theta_{1}}{\theta_{j}}\phi(x_{i_{0}1}).

By balancedness, there is some i1i_{1} so that xi1​1=xi0​jx_{i_{1}1}=x_{i_{0}j}, so repeating the above argument, we get ϕ​(xi1​j)=βˆ’ΞΈ1ΞΈj​ϕ​(xi1​1)=βˆ’ΞΈ1ΞΈj​ϕ​(xi0​j)=(βˆ’ΞΈ1ΞΈj)2​ϕ​(xi0​1)\phi(x_{i_{1}j})=-\frac{\theta_{1}}{\theta_{j}}\phi(x_{i_{1}1})=-\frac{\theta_{1}}{\theta_{j}}\phi(x_{i_{0}j})=(-\frac{\theta_{1}}{\theta_{j}})^{2}\phi(x_{i_{0}1}). Iterating, this argument shows that (βˆ’ΞΈ1ΞΈj)m​ϕ​(xi0​1)∈{ϕ​(x):x∈X}(-\frac{\theta_{1}}{\theta_{j}})^{m}\phi(x_{i_{0}1})\in\{\phi(x):x\in X\} for all mβˆˆβ„€m\in\mathbb{Z}, implying that βˆ’ΞΈ1ΞΈj-\frac{\theta_{1}}{\theta_{j}} is a root of unity.

Now let Ο‰1=1\omega_{1}=1 and Ο‰j=βˆ’ΞΈ1ΞΈj\omega_{j}=-\frac{\theta_{1}}{\theta_{j}} for j>1j>1. Dividing the equation (10.1) with i=i0i=i_{0} by xi0​1x_{i_{0}1} and applying Ο•\phi to both sides, we have βˆ‘i=1nϕ​(ai)​ωi=0\sum_{i=1}^{n}\phi(a_{i})\omega_{i}=0. Finally, letting ρi=Οˆβˆ’1​(Ο‰i)\rho_{i}=\psi^{-1}(\omega_{i}), we see that Οˆβ€‹(βˆ‘i=1nai​ρi)=βˆ‘i=1nϕ​(ai)​ωi=0\psi(\sum_{i=1}^{n}a_{i}\rho_{i})=\sum_{i=1}^{n}\phi(a_{i})\omega_{i}=0, and hence βˆ‘i=1nai​ρi=0\sum_{i=1}^{n}a_{i}\rho_{i}=0. ∎

The above work, along with Lemma 4.3, reduces Conjecture 10.1 to the following.

Conjecture 10.7.

Let KK be a number field and π’ͺK{\mathcal{O}}_{K} its ring of integers. If (a1,…,an)∈π’ͺKn(a_{1},...,a_{n})\in{\mathcal{O}}_{K}^{n} satisfies the strong absolute value criteria, then (a1,…,an)(a_{1},...,a_{n}) is a Smyth tuple.

Speyer [1] gives a proof of Conjecture 10.7 in the case where n=3n=3 and a1=a2a_{1}=a_{2}.

Speyer’s argument works for general nn and a1=…=anβˆ’1a_{1}=...=a_{n-1} with minimal modification; this result is our final proposition.

Proposition 10.8.

Let nβ‰₯3n\geq 3 be an integer. Let KK be a number field and π’ͺK{\mathcal{O}}_{K} its ring of integers. Let α∈π’ͺK\alpha\in{\mathcal{O}}_{K} so that every archimedean absolute value of Ξ±\alpha is less than nβˆ’1n-1. Then (1,1,…,1,Ξ±)∈π’ͺKn(1,1,\dots,1,\alpha)\in{\mathcal{O}}_{K}^{n} is a Smyth tuple.

Proof.

By Lemma 10.3, it suffices to show (1,1,…,1,βˆ’Ξ±)(1,1,\dots,1,-\alpha) is a Smyth tuple. By Proposition 4.1, it suffices for us to show that there are permutation matrices XiX_{i} so that βˆ‘i=1nβˆ’1Xi\sum_{i=1}^{n-1}X_{i} has Ξ±\alpha as an eigenvalue.

We will follow the argument from [1], starting with a slight generalization of Speyer’s Step 1, which we write out in full for the sake of clarity.

Step 1: There is a nonnegative integer matrix CC whose rows sum to nβˆ’1n-1 with eigenvalue Ξ±\alpha.

Consider the lattice A=℀​[Ξ±]A={\mathbb{Z}}[\alpha] and the vector space V=AβŠ—β„€β„V=A\otimes_{\mathbb{Z}}{\mathbb{R}}. Since Ξ±\alpha is an algebraic integer, AA is a discrete full sublattice of VV. We take the norm βˆ‘Ξ½|x|Ξ½2\sum_{\nu}|x|_{\nu}^{2}, where the sum runs over all archimedean places. Let c=maxν⁑|Ξ±|Ξ½c=\max_{\nu}|\alpha|_{\nu}. By hypothesis, c<nβˆ’1c<n-1. Denote by BRB_{R} the closed ball of radius RR around 0.

Let MM be large enough so that any ball of radius MM around any point in VV contains a point in AA. Take RR large enough so that cnβˆ’1​R+(nβˆ’2)​M<R\frac{c}{n-1}R+(n-2)M<R. Now, for any z∈A∩BRz\in A\cap B_{R}, let z1∈A∩BRz_{1}\in A\cap B_{R} be the closest point to α​znβˆ’1\frac{\alpha z}{n-1}. Let z2=α​zβˆ’(nβˆ’2)​z1z_{2}=\alpha z-(n-2)z_{1}. Now,

|z1|≀|z1βˆ’Ξ±β€‹znβˆ’1|+|α​znβˆ’1|≀M+cnβˆ’1​R<R|z_{1}|\leq|z_{1}-\frac{\alpha z}{n-1}|+|\frac{\alpha z}{n-1}|\leq M+\frac{c}{n-1}R<R

Similarly,

|z2|=|α​zβˆ’(nβˆ’2)​z1|≀|α​zβˆ’nβˆ’2nβˆ’1​α​z|+(nβˆ’2)​|α​znβˆ’1βˆ’z1|≀cnβˆ’1​R+(nβˆ’2)​M<R|z_{2}|=|\alpha z-(n-2)z_{1}|\leq|\alpha z-\frac{n-2}{n-1}\alpha z|+(n-2)|\frac{\alpha z}{n-1}-z_{1}|\leq\frac{c}{n-1}R+(n-2)M<R

Thus, for any z∈A∩BRz\in A\cap B_{R}, we can find z1,z2∈A∩BRz_{1},z_{2}\in A\cap B_{R} so that (nβˆ’2)​z1+z2=z(n-2)z_{1}+z_{2}=z. Enumerate the elements of A∩BRA\cap B_{R} as z1,z2,…,zlz_{1},z_{2},\dots,z_{l}. Then, we can form an lΓ—ll\times l matrix CC with the following entries. For the ii-th row, consider ziz_{i}. As before, we may write (nβˆ’2)​zj+zk=zi(n-2)z_{j}+z_{k}=z_{i} for some 1≀j,k≀l1\leq j,k\leq l. In the ii-th row, put nβˆ’2n-2 in the jj-th column and 11 in the kk-th column if jβ‰ kj\neq k; if j=kj=k, put an nβˆ’1n-1 in the jt​hj^{th} column. This matrix CC has all row sums nβˆ’1n-1. By construction, it has Ξ±\alpha as an eigenvalue with right eigenvector (z1,z2,…,zl)T(z_{1},z_{2},\dots,z_{l})^{T}.

The rest of Speyer’s argument can now be applied with virtually no modification; using the Perron-Frobenius theorem, one obtains a matrix DD from CC which is the sum of nβˆ’1n-1 permutation matrices and still has Ξ±\alpha as an eigenvalue. ∎

References

  • [1] David E Speyer (https://mathoverflow.net/users/297/david-e-speyer) β€œTell me an algebraic integer that isn’t an eigenvalue of the sum of two permutations” URL:https://mathoverflow.net/q/264035 (version: 2017-03-09), MathOverflow URL: https://mathoverflow.net/q/264035
  • [2] N. Berry et al. β€œThe conjugate dimension of algebraic numbers” In The Quarterly Journal of Mathematics 55.3, 2004, pp. 237–252 DOI: 10.1093/qmath/hah003
  • [3] H.B. Mann β€œOn linear relations between roots of unity” In Mathematika 12, 1965, pp. 107–117
  • [4] C.J. Smyth β€œAdditive and Multiplicative Relations Connecting Conjugate Algebraic Numbers” In Journal of Number Theory, 1986, pp. 243–254 DOI: https://doi.org/10.1016/0022-314X(86)90094-6
  • [5] U. Zannier β€œVanishing sums of roots of unity” In Rend. Sem. Mat. Univ. Pol. Torino 53, 4, 1995, pp. 487–495